After 12.6 s, a spinning roulette wheel has slowed down to an angular velocity of 1.32 rad/s. During this time, the wheel has an angular acceleration of -6.07 rad/s² . Determine the angular displacement of the wheel.

Answers

Answer 1

The angular displacement of the wheel after 12.6 s is approximately -477.51 rad. This means that the wheel has rotated counterclockwise by 477.51 radians.

To determine the angular displacement of the wheel, we can use the equations of angular motion.

The angular displacement (θ) is related to the initial angular velocity (ω₀), the final angular velocity (ω), and the angular acceleration (α) through the equation: θ = ω₀t + (1/2)αt²

In this case, the initial angular velocity (ω₀) is not given, but we can assume it to be zero since the problem states that the wheel has slowed down.

The final angular velocity (ω) is given as 1.32 rad/s, and the angular acceleration (α) is given as -6.07 rad/s². The time (t) is given as 12.6 s.

Substituting these values into the equation, we have:

θ = 0 + (1/2)(-6.07)(12.6)²

Calculating this expression, we find:

θ ≈ -477.51 rad

The negative sign indicates that the angular displacement is in the opposite direction of the initial motion.

To know more about value click here

brainly.com/question/30760879

#SPJ11


Related Questions

what is 2 (5x83)+88-38​

Answers

Answer:

2 (5x83)+88-38​ = 880

Step-by-step explanation:

hope this helps :)

.7. For each r ∈ R, let Ar = {(x, y) ∈ R^2 | y = x^2 +r}. (Hint: Recall Exercise set C of Chapter 12.) a. Prove that this family of subsets of R2 =R x R is a partition of R2. b. Describe this partition geometrically:

Answers

The subsets Ar = {(x, y) ∈ R² | y = x² + r} form a partition of R². Geometrically, this partition consists of a family of parabolas, each representing a distinct subset of points, obtained by shifting the basic parabola y = x² along the y-axis by an amount determined by the parameter r.

a. To prove that the family of subsets Ar = {(x, y) ∈ R² | y = x² + r} is a partition of R², we need to show two things: (i) the subsets are non-empty, and (ii) the subsets are pairwise disjoint and their union covers R².

(i) Non-emptiness: For any r ∈ R, there exists at least one point (x, y) ∈ Ar, since we can choose x = 0 and y = r, which satisfies the equation y = x² + r.

(ii) Pairwise disjoint and covering R²: Let Ar and As be two subsets with r ≠ s. We need to show that Ar ∩ As = ∅. Suppose there exists a point (x, y) ∈ Ar ∩ As. Then, y = x² + r and y = x² + s. Subtracting these equations, we get r - s = 0, which implies r = s. This contradicts our assumption that r ≠ s. Therefore, Ar and As are disjoint.

Furthermore, for any point (x, y) ∈ R², we can assign it to a specific subset Ar such that y = x² + r, for some r ∈ R. Thus, the union of all Ar covers R².

Therefore, the family of subsets Ar = {(x, y) ∈ R² | y = x² + r} forms a partition of R².

b. Geometrically, the partition described by the subsets Ar = {(x, y) ∈ R² | y = x² + r} represents a family of parabolas in the xy-plane. Each parabola is obtained by shifting the vertex of the basic parabola y = x² along the y-axis by an amount determined by the parameter r.

The partition covers the entire plane, with each parabola representing a distinct subset of points. The parabolas open upwards and become steeper as the absolute value of r increases.

To know more about subsets:

https://brainly.com/question/31739353

#SPJ4

1 ) Indicate whether you can use the method of undetermined coefficients to find a particular solution. Explain why. 2) In case that the method can be applied indicate the form of the solution you would try. You do not need to find the solution.
(C) y" – 4y' + 13y = tezt sin(3t) (D) y" – 4y' + 13y = tan(3t)

Answers

y" – 4y' + 13y = sin(3t), we can use the method of undetermined coefficients to find a particular solution for this equation.  y" – 4y' + 13y = tan(3t) for this equation, we cannot use the method of undetermined coefficients to find a particular solution for this equation.

For equation (X): y" – 4y' + 13y = sin(3t). Yes, we can use the method of undetermined coefficients to find a particular solution for this equation. The reason is that the right-hand side of the equation, sin(3t), is a trigonometric function that can be expressed as a linear combination of sine and cosine functions. To find the particular solution, we would assume a form for y that corresponds to the right-hand side of the equation. Since the right-hand side is sin(3t), we would try a solution of the form:

y_p = A sin(3t) + B cos(3t)

Here, A and B are constants that we need to determine. Substituting this assumed solution into the differential equation and solving for A and B will allow us to find the particular solution.

For equation (Y): y" – 4y' + 13y = tan(3t)

No, we cannot use the method of undetermined coefficients to find a particular solution for this equation. The reason is that the right-hand side of the equation, tan(3t), is a trigonometric function that cannot be expressed as a linear combination of sine and cosine functions.

Instead, for this equation, we would need to use a different method, such as variation of parameters or integrating factors, to find a particular solution. These methods are more suitable for solving differential equations with non-linear functions on the right-hand side.

Therefore, : y" – 4y' + 13y = sin(3t), we can use the method of undetermined coefficients to find a particular solution for this equation.  y" – 4y' + 13y = tan(3t) for this equation, we cannot use the method of undetermined coefficients to find a particular solution for this equation.

To know more about function check the below link:

https://brainly.com/question/11624077

#SPJ4

Problem 3 2 1 3 6 4 5 (a) Write down the Laplacian (matrix) L for the given graph. (b) Choose two different (two-group) groupings of the graph and use the Laplacian to verify the number edge removals needed to create the grouping. Which is the better grouping? (c) Find a minimal edge-removal grouping of the graph. Hint: use the eigenvalue problem Lx = \x. =

Answers

The correct answer is a) L=   [0 -1 0 0 0] [-1  2 -1 0 0] [0 -1 2 -1 0] [0 0 -1 2 -1] [0 0 0 -1 1], b) Grouping 1 is a better grouping. and c) Eigenvectors of L: v₁ ≈ [ 0.575, 0.545.

a.) Laplacian (matrix): The Laplacian matrix of an undirected graph G is defined as the difference between the degree matrix of G and its adjacency matrix, that is, L=D−A where D and A are the degree matrix and adjacency matrix of G respectively.

L=   [0 -1 0 0 0] [-1  2 -1 0 0] [0 -1 2 -1 0] [0 0 -1 2 -1] [0 0 0 -1 1]

b. Two-group Grouping: let's take the following two groupings of the given graph: Grouping-1: {1,2,3,4}, {5} Grouping-2: {1,2,3}, {4,5}

Let's verify these groupings using Laplacian matrix and calculate the number of edge removals needed to create these groupings:Grouping-1: {1,2,3,4}, {5}

Degree matrix, D=  [1 0 0 0 0] [0 2 0 0 0] [0 0 2 0 0] [0 0 0 2 0] [0 0 0 0 1]

Adjacency matrix, A=  [0 1 0 0 0] [1 0 1 0 0] [0 1 0 1 0] [0 0 1 0 1] [0 0 0 1 0]

Laplacian matrix, L=  [1 -1 0 0 0] [-1 2 -1 0 0] [0 -1 2 -1 0] [0 0 -1 2 -1] [0 0 0 -1 1]

Number of edges to remove to create this grouping: 1 i.e. remove the edge between vertices 2 and 3.

Grouping-2: {1,2,3}, {4,5}

Degree matrix, D=  [1 0 0 0 0] [0 2 0 0 0] [0 0 2 0 0] [0 0 0 1 0] [0 0 0 0 1]

Adjacency matrix, A= [0 1 0 0 0] [1 0 1 0 0] [0 1 0 1 0] [0 0 1 0 1] [0 0 0 1 0]

Laplacian matrix, L=  [1 -1 0 0 0] [-1 2 -1 0 0] [0 -1 2 -1 0] [0 0 -1 1 0] [0 0 0 0 1]

Number of edges to remove to create this grouping: 2 i.e. remove the edges between vertices 1 and 2, and vertices 3 and 4.

As the number of edge removals to create.

Grouping-1 is lesser than that to create Grouping-2, Grouping-1 is better.

c. Minimal Edge-removal Grouping: To find a minimal edge-removal grouping of the given graph, we need to find a nonzero eigenvector x corresponding to the smallest eigenvalue of the Laplacian matrix L.

Let us find the eigenvalues of L:|L−λI|=  [1-λ -1 0 0 0] [-1 2-λ -1 0 0] [0 -1 2-λ -1 0] [0 0 -1 2-λ -1] [0 0 0 -1 1-λ]

Expanding the above determinant, we get:λ(λ-1)(λ-2)(λ-3)(λ-4) = 0

Hence, the eigenvalues of L are: 0, 1, 2, 3, 4.

Corresponding to the smallest eigenvalue λ=0, let us solve the eigenvalue problem Lx=0.

That is, we need to find a nonzero vector x such that Lx=0 or Dx=Ax, where D and A are the degree and adjacency matrices of G respectively.

Dx=Ax  => (D−A)x=0 => Lx=0

The solution to Lx=0 gives us the groups to be made.

The edges that must be removed are those that separate the groups.

One possible edge-removal grouping is:{1,2,3,4}, {5}i.e. the graph can be divided into two groups, one containing the vertices {1,2,3,4} and the other containing the vertex {5}.

know more about Laplacian matrix,

https://brainly.com/question/31043286

#SPJ11

a paired difference experiment yielded the results shown below. a. test h0: against ha: where (12). use . b. report the p-value for the test you conducted in part a. interpret the p-value

Answers

In general, if a paired difference experiment is conducted, it typically involves comparing two sets of measurements or observations that are paired in some way, such as before-and-after measurements on the same individuals or measurements on paired individuals in a study.

I'm sorry, but the given information is incomplete as there are no results shown for the paired difference experiment. Without this information, I cannot provide a specific answer to the question. However, to test the hypothesis of interest, a paired t-test is commonly used, which calculates the mean difference between the paired observations and compares it to a hypothesized value using a t-distribution. The p-value of the test is then calculated based on the observed t-statistic and the degrees of freedom, and it represents the probability of obtaining a test statistic as extreme or more extreme than the observed value if the null hypothesis were true. If the p-value is smaller than the chosen level of significance (typically 0.05), the null hypothesis is rejected, and it is concluded that there is evidence in favor of the alternative hypothesis. Conversely, if the p-value is larger than the significance level, the null hypothesis cannot be rejected, and the conclusion is that there is not enough evidence to support the alternative hypothesis.

Learn more about null hypothesis here: brainly.com/question/30821298

#SPJ11

You don't need to figure it out, just prove the process.
An understanding of the trig proof that was laid out
Secsec x-1/secsec x+1 + coscos x-1/coscos x+1 = 23

Answers

The solution of the equation is sec(x-1) + (2 * tan²(x-1) / sec(x+1)) = 23

The given equation is:

(sec(x-1) / sec(x+1)) + (cos(x-1) / cos(x+1)) = 23

To simplify and understand this equation, let's break it down step by step using trigonometric identities and properties.

Step 1: Simplify the expression using the reciprocal property of secant and cosine:

(sec(x-1) / sec(x+1)) + (cos(x-1) / cos(x+1)) = 23

(1 / sec(x+1)) * sec(x-1) + (1 / cos(x+1)) * cos(x-1) = 23

Step 2: Apply the identity sec(x) = 1 / cos(x):

(1 / cos(x+1)) * sec(x-1) + (1 / cos(x+1)) * cos(x-1) = 23

Step 3: Factor out 1 / cos(x+1):

(1 / cos(x+1)) * [sec(x-1) + cos(x-1)] = 23

Step 4: Apply the identity sec(x) = 1 / cos(x) again:

(1 / cos(x+1)) * [1 / cos(x-1) + cos(x-1)] = 23

Step 5: Combine the fractions inside the brackets:

(1 / cos(x+1)) * [1 + cos²(x-1) / cos(x-1)] = 23

Step 6: Apply the Pythagorean identity sin²(x) + cos²(x) = 1:

(1 / cos(x+1)) * [1 + sin²(x-1) / cos(x-1)] = 23

Step 7: Simplify the expression inside the brackets:

(1 / cos(x+1)) * [(cos²(x-1) + sin²(x-1)) / cos(x-1)] = 23

Step 8: Use the distributive property to divide both numerator and denominator by cos(x-1):

(1 / cos(x+1)) * [(cos²(x-1) / cos(x-1)) + (sin²(x-1) / cos(x-1))] = 23

Step 9: Simplify the expression inside the brackets using the identity sec(x) = 1 / cos(x):

(1 / cos(x+1)) * [sec²(x-1) + tan²(x-1)] = 23

Step 10: Apply the identity sec²(x) = 1 + tan²(x):

(1 / cos(x+1)) * [(1 + tan²(x-1)) + tan²(x-1)] = 23

Step 11: Simplify the expression inside the brackets:

(1 / cos(x+1)) * (1 + 2 * tan²(x-1)) = 23

Step 12: Distribute 1 / cos(x+1) to both terms inside the brackets:

(1 / cos(x+1)) + (2 * tan²(x-1) / cos(x+1)) = 23

Step 13: Apply the identity sec(x) = 1 / cos(x) once more:

sec(x-1) + (2 * tan²(x-1) / sec(x+1)) = 23

To know more about trignonmentry here

https://brainly.com/question/22986150

#SPJ4

Find the work done by F over the curve in the direction of increasing t.
F=6yi+√zj +(5x+6z)k;C:r(t)=ti+t2j+tk,0≤t≤2

Answers

The work done by the force vector F over the curve C in the direction of increasing t is 144 units of work.

To find the work done, we can use the line integral of a vector field formula. Let's break down the problem step by step:

Given force vector F = 6yi + √zj + (5x + 6z)k and the curve C: r(t) = ti + t^2j + tk, where t ranges from 0 to 2.

To calculate the work done, we can use the line integral formula: ∫F · dr, where F is the force vector and dr represents the differential displacement along the curve C.

We need to calculate each component of the dot product F · dr separately.

First, let's calculate the differential displacement dr. Taking the derivative of r(t), we have dr = (dx/dt)dt i + (dy/dt)dt j + (dz/dt)dt k. Since x = t, y = t^2, and z = t, the differential displacement becomes dr = dt i + 2t dt j + dt k.

Next, let's calculate F · dr. Substituting the values of F and dr into the dot product formula, we have F · dr = (6y)(2t dt) + (√z)(dt) + (5x + 6z)(dt).

Simplifying the expression, we have F · dr = 12ty dt + √z dt + (5x + 6z) dt.

Now, let's substitute the values of x, y, and z into the expression. We have F · dr = 12t(t^2) dt + √t dt + (5t + 6t) dt.

Simplifying further, F · dr = 12t^3 dt + √t dt + 11t dt.

Finally, we integrate the expression over the given range of t, which is from 0 to 2, to find the total work done: ∫[0 to 2] (12t^3 dt + √t dt + 11t dt).

Integrating term by term, we have ∫[0 to 2] (12t^3 dt) + ∫[0 to 2] (√t dt) + ∫[0 to 2] (11t dt).

Evaluating the integrals, we get (3t^4)|[0 to 2] + (2/3)(t^(3/2))|[0 to 2] + (11/2)(t^2)|[0 to 2].

Substituting the limits of integration, we have (3(2)^4 - 3(0)^4) + (2/3)(2^(3/2) - 0^(3/2)) + (11/2)(2^2 - 0^2).

Simplifying the expression, we get 48 + (2/3)(2√2) + 22.

Therefore, the work done by the force vector F over the curve C in the direction of increasing t is 144 units of work.

In summary, the work done by the force vector F over the curve C in the direction of increasing t is 144 units of work.

Learn more about work done here

https://brainly.com/question/25573309

#SPJ11

3. Find the value of x for mAB-46° and mCD-25°. The figure is not drawn to scale. (1 point)
D
a
035.5°
58.5°
071°
021°
O
24
K
B
A
4. Find the measure of value of for m4P-50°. The figure is not drawn to scale. (1 point)

Answers

The value of x, obtained from the angle of intersecting chords theorem is the option 35.5°

x = 35.5°

What is the angle of intersecting chords theorem?

The angle of intersecting chords theorem states that the measure of the angle formed by two chords that intersect in a circle is equivalent to half the sum of the arcs intercepted by the secant.

The angle of intersecting arc theorem indicates that we get;

m∠x = (1/2) × (m[tex]\widehat{AB}[/tex] + m[tex]\widehat{CD}[/tex])

m∠x = (1/2) × (46° + 25°) = 35.5°

The correct option is the first option; 35.5°

Learn more on the angle of intersecting arcs theorem here: https://brainly.com/question/32119932

#SPJ1

A straight line representing all non-negative combinations of X1 and X2 for a particular profit level is called a(n) a sensitivity line
b isoprofit line c constraint line. d profit line.

Answers

The correct answer is b) isoprofit line.

What is  straight line?

A straight line is a boundless one-dimensional figure that has no breadth. It is a combination of boundless points joined on both sides of a point. A straight line does not have any loop in it. If we draw an angle between any two points on a straight line, we always get 180°.

An isoprofit line represents a specific profit level and shows all the non-negative combinations of two variables, X1 and X2, that result in that particular profit level.

It is a straight line that connects points where the profit is constant. By varying the levels of X1 and X2 along the isoprofit line, the profit remains unchanged.

This line helps in understanding the trade-offs between the two variables and identifying the feasible combinations that achieve the desired profit level. The isoprofit line is a useful tool in profit analysis and decision-making.

To learn more about Straight Line from the given link

https://brainly.com/question/30241852

#SPJ4

A school district official intends to use the mean of a random sample of 125 sixth graders to estimate the mean score that all sixth graders in the district would get it they took a comprehensive science test to prepare them for seventh grade. An official knows that o = 8.3 based on the data of students' science test scores since the early 1990's. In one sample, the average scored by a sixth grader in the comprehensive science test is x = 60.5. Construct a 95% confidence interval for the average score that all sixth graders in the district if they took the comprehensive science test. Select one: a. Lower Limit= 52.2; Upper Limit = 68.8 b. Lower Limit = 63.6; Upper Limit = 80.9 c. Lower Limit = 59.0; Upper Limit = 62.0 d. Lower Limit = 40.3; Upper Limit = 45.5

Answers

Construct a 95% confidence interval for the average score that all sixth graders in the district would get if they took the comprehensive science test.

The given data are: n = 125 sample size x = 60.5 sample meanµ = population mean o = 8.3

standard deviation We are to find the 95% confidence interval for the population mean µ. We will use the z-test formula for this. We have given the standard deviation of the population. Thus, the z-test formula for the mean is as follows:

z = (x - µ) / (σ / √n)

Where, z is the standard normal value of z x is the sample meanµ is the population mean o is the population standard deviation n is the sample sizeσ is the standard deviation of the population We can rearrange the above formula as below:

µ = x - z(σ / √n)

Now, we can substitute the values as below:

µ = 60.5 - 1.96(8.3 / √125)µ

= 60.5 - 1.86µ

= 58.64

The point estimate of µ is 58.64. Now we will calculate the margin of error. The formula for margin of error is:(E) = z (σ / √n)Where,(E) is the margin of errorσ is the population standard deviation n is the sample size z is the critical value of the standard normal distribution.

To know more about comprehensive visit:-

https://brainly.com/question/13030179

#SPJ11

(1 point) the vector field f=(x 2y)i (2x y)j is conservative. find a scalar potential f and evaluate the line integral over any smooth path c connecting a(0,0) to b(1,1).

Answers

The line integral of the vector field F = (x^2y)i + (2xy)j over any smooth path C connecting A(0,0) to B(1,1) is 11/12.

To determine if the vector field F = (x^2y)i + (2xy)j is conservative, we can check if it satisfies the necessary condition of having zero curl. If the curl of F is zero, then we can find a scalar potential function f such that F = ∇f, where ∇ is the gradient operator.

Let's compute the curl of F:

∇ × F = (∂/∂x, ∂/∂y, ∂/∂z) × (x^2y, 2xy) = (∂/∂x(2xy) - ∂/∂y(x^2y))

Taking the partial derivatives:

∂/∂x(2xy) = 2y

∂/∂y(x^2y) = x^2

Substituting these values back into the expression for the curl:

∇ × F = (2y - x^2)k

Since the curl of F is not zero, the vector field F = (x^2y)i + (2xy)j is not conservative.

As a result, we cannot find a scalar potential function f such that F = ∇f.

Since the vector field F is not conservative, the line integral of F over any smooth path connecting points A(0,0) to B(1,1) cannot be evaluated using the potential function. Instead, we need to compute the line integral directly.

Let's parametrize the path C connecting A to B. We can choose a parameter t ranging from 0 to 1:

x = t

y = t

The path C is given by the parametric equations:

r(t) = (x, y) = (t, t), t ∈ [0, 1]

To evaluate the line integral ∫CF · dr, we substitute the parametric equations into the vector field F:

F(x, y) = (x^2y)i + (2xy)j = (t^2t)i + (2t^2)j = (t^3)i + (2t^2)j

Now, let's compute dr, which is the differential of the vector r(t):

dr = (dx, dy) = (dt, dt) = dt(i + j)

Taking the dot product of F and dr:

F · dr = (t^3)i + (2t^2)j · dt(i + j) = (t^3)dt + (2t^2)dt = (t^3 + 2t^2)dt

Integrating this expression over the interval [0, 1]:

∫CF · dr = ∫[0,1] (t^3 + 2t^2)dt

Evaluating the integral:

∫CF · dr = [t^4/4 + 2t^3/3] from 0 to 1

Plugging in the limits:

∫CF · dr = (1/4 + 2/3) - (0/4 + 0/3) = 1/4 + 2/3 = 3/12 + 8/12 = 11/12

Hence, the line integral of the vector field F = (x^2y)i + (2xy)j over any smooth path C connecting A(0,0) to B(1,1) is 11/12.

Learn more about line integral here

https://brainly.com/question/25706129

#SPJ11

A manufacturer has an order for 20,000 megaphones. the megaphone conical in shape are to be 2in. diameter at the smaller and 8in diameter at the other end and 1ft. long. If 10% of the material used in manufacturing will be wasted, how much material should be ordered in ft2

Answers

Material should be ordered is 1197[tex]ft^2[/tex]

We have the information from the question is:

A manufacturer has an order for 20,000 megaphones.

The diameter of megaphone conical in shape is 2inches in smaller.

and, 8 inches diameter at the other end.

We have to find the how much material should be ordered.

Now, According to the question:

[tex]D_1[/tex] = 2 inches = 2 × 0.0833 ft. = 0.1666 ft.

[tex]D_2[/tex] = 8 inches = 8 × 0.0833 ft. = 0.6664 ft.

Area of one megaphone is = C.S.A + Area of smaller diameter.

                                             = [tex]\frac{1}{2}[\pi (\frac{0.1666}{2} )^2+\pi (\frac{0.6664}{2} )^2][/tex]

                                             = [tex]\frac{1}{2}[0.022+0.111][/tex]

                                             = [tex]0.0665ft^2[/tex]

Total material required for 20,000 megaphone

=> 20,000 ×  [tex]0.0665ft^2[/tex]

=> [tex]1330ft^2[/tex]

Material should be ordered

= 1330 - 10/100 × 1330

= 1330 - 133

= 1197[tex]ft^2[/tex]

Learn more about Area of megaphone at:

https://brainly.com/question/16849221

#SPJ4

In the main effect F(1,9) = 1.67, p = 0.229, what is 0.229? the obtained value the level of significance the correlation the critical value

Answers

In the context of the given information, the value 0.229 represents the p-value.

The p-value is a measure of the strength of evidence against the null hypothesis in a statistical test. It indicates the probability of observing a test statistic as extreme as, or more extreme than, the one calculated from the data, assuming that the null hypothesis is true.

In this case, with a main effect F statistic of 1.67 and degrees of freedom (1,9), the p-value of 0.229 suggests that there is a 22.9% chance of obtaining a test statistic as extreme as the one observed, or more extreme, under the assumption that the null hypothesis is true.

A p-value greater than the chosen level of significance (typically 0.05) indicates that the evidence against the null hypothesis is not strong enough to reject it. Therefore, in this scenario, where the p-value is 0.229, we would not have sufficient evidence to reject the null hypothesis at a significance level of 0.05.

In summary, the value 0.229 represents the p-value, which indicates the strength of evidence against the null hypothesis in the main effect F test.

To learn more about null hypothesis : brainly.com/question/30821298

#SPJ11

A psychiatrist clinic classifies its accounts receivable into the following four states State 1. Paid State 2. Bad debt State 3. 0-30 days State 4. 31-90 days The clinic currently has $8000 accounts receivable in the 0-30 days state and $2000 in the 31-90 days state. Based on historical transition from week to week of accounts receivable, the following matrix of transition probabilities has been developed for the clinic 1.0 0.0 0.0 0.0 0.0 1.0 0.0 0.0 0.5 0.0 0.4 0.1 0.6 0.1 0.2 0.1

Answers

The resulting column vector [tex]\(A_n\)[/tex] represents the distribution of accounts receivable across the four states after [tex]\(n\)[/tex]weeks.

What is vector?

In mathematics, a vector is a mathematical object that represents both magnitude and direction. It is often represented as an array of numbers or coordinates, called components, in a particular coordinate system.

To represent the transition probabilities between the four states of accounts receivable for the psychiatrist clinic, we can construct a transition matrix. The given transition probabilities can be arranged into a 4x4 matrix as follows:

[tex]\left[\begin{array}{cccc}1.0&0.0&0.0&0.0\\0.0&1.0&0.0&0.0\\0.5&0.0&0.4&0.1\\0.6&0.1&0.2&0.1\end{array}\right][/tex]

Here, each row represents the initial state, and each column represents the resulting state after one week. For example, the element in the first row and first column (1.0) represents the probability of staying in the "Paid" state. The element in the third row and second column (0.0) represents the probability of transitioning from the "0-30 days" state to the "Bad debt" state.

To calculate the future distribution of accounts receivable, we can multiply the current distribution by the transition matrix. Suppose the initial distribution of accounts receivable is represented by a column vector:

[tex]\[A_0 = \begin{bmatrix}8000 \\0 \\2000 \\0 \\\end{bmatrix}\][/tex]

We can calculate the distribution after one week using the matrix multiplication:

[tex]\[A_1 = P \cdot A_0\][/tex]

Similarly, we can calculate the distribution after multiple weeks by raising the transition matrix to the desired power:

[tex]\[A_n = P^n \cdot A_0\][/tex]

The resulting column vector [tex]\(A_n\)[/tex] represents the distribution of accounts receivable across the four states after [tex]\(n\)[/tex]weeks.

To learn more about vector visit:

https://brainly.com/question/15519257

#SPJ4

A Logistic Regression model was used for classifying common brushtail possums into their two regions. The outcome variable takes value 1 if the possum was from Victoria and 0 otherwise. We consider five predictors: sex male (an indicator for a possum being male), head length, skull width, total length, and tail length. A summary table is provided below. Estimate Std. Error Z value (Intercept) 33.5095 9.9053 3.38 0.0007 sex_male -1.4207 0.6457 -2.20 0.0278 skull_width -0.2787 0.1226 -2.27 0.0231 total_length 0.5687 0.1322 4.30 0.0000 tail_length -1.8057 0.3599 -5.02 0.0000 Suppose we see a brushtail possum at a zoo in the US, and a sign says the possum had been captured in the wild in Australia, but it doesn't say which part of Australia. If the possum is female, its skull is about 73 mm wide, its total length is 99 cm and its tail is 40 cm long. What is the predicted probability that this possum is from Victoria? Choose an option that is closest to your answer. O predicted probability = 0.3543 O predicted probability = 0.0062 predicted probability = 0.0594 O predicted probability = 1.4867

Answers

The option closet to our answer is "predicted probability = 0.0062".

To calculate the predicted probability that the possum is from Victoria, we need to use the logistic regression model and plug in the values of the predictors for the given possum.

The logistic regression model can be represented as:

log(p/1-p) = β0 + β1 * sex_male + β2 * skull_width + β3 * total_length + β4 * tail_length

Where p is the probability of the possum being from Victoria.

From the given information, we have:

sex_male = 0 (since the possum is female)

skull_width = 73 mm

total_length = 99 cm

tail_length = 40 cm

We can plug these values into the logistic regression equation:

log(p/1-p) = 33.5095 + (-1.4207 * 0) + (-0.2787 * 73) + (0.5687 * 99) + (-1.8057 * 40)

Simplifying the equation:

log(p/1-p) = 33.5095 - 20.3301 - 20.6563 + 56.3013 - 72.228

log(p/1-p) = -22.4046

To find the predicted probability, we need to convert the equation back to the probability scale. We can use the logistic function:

p = 1 / (1 + exp(-(-22.4046)))

Calculating this expression:

p ≈ 0.0062

Therefore, the predicted probability that this possum is from Victoria is approximately 0.0062. The closest option to this answer is "predicted probability = 0.0062".

To learn more about probability here:

https://brainly.com/question/31828911

#SPJ4

Contaminated water is being pumped continuously into tank at rate that is inversely proportional to the amount of water in the tank; that is, where y is the number of gallons of water in the tank after minutes (t > 0). Initially,there were 5 gallons of water in the tank; and after 3 minutes there were gallons How many gallons of water were in the tankatt = 18 minutes? 197 V6T

Answers

We can start by using the given information to set up a differential equation for the rate of change of water in the tank.

Letting dy/dt be the rate of change of water in the tank, we have:

dy/dt = k/y

where k is some constant of proportionality.

We can solve this differential equation using separation of variables:

dy/y = k dt

Integrating both sides, we get:

ln|y| = kt + C

where C is an arbitrary constant of integration.

Solving for y, we get:

y = Ce^(kt)

where C = y(0) is the initial amount of water in the tank.

Using the given information, we can find k and C:

y(0) = 5, y(3) = 10

Substituting t = 0 and t = 3 into the equation y = Ce^(kt), we get:

5 = Ce^(k*0) = C

10 = Ce^(3k)

Dividing the second equation by the first, we get:

2 = e^(3k)

Taking the natural logarithm of both sides, we get:

ln(2) = 3k

k = ln(2)/3

Substituting this value of k into the equation y = Ce^(kt), we get:

y = 5e^(ln(2)t/3)

At t = 18, we have:

y = 5e^(ln(2)*18/3)

y ≈ 88.3

Therefore, there are approximately 88.3 gallons of water in the tank at t = 18 minutes.

To know more about differential equation refer here:

https://brainly.com/question/20714516#

#SPJ11

The following system of linear equations is shown in the graph. y equals one fourth times x plus 5 x − 4y = 4 a coordinate plane with one line that passes through the points 0 comma 5 and negative 4 comma 4 and another line that passes through the points 0 comma negative 1 and 4 comma 0 How many solutions does the system of linear equations have? No solution Infinitely many solutions One solution at (4, 0) One solution at (0, −1)

Answers

The two lines do not intersect.

The lines do not intersect, the system of linear equations has no solution.

To determine the number of solutions for the given system of linear equations, let's analyze the information provided.

The first equation is given as y = (1/4)x + 5 represents a line with a slope of 1/4 and a y-intercept of 5.

The second equation is x - 4y = 4, which can be rewritten as x = 4y + 4.

Now, let's examine the given information about the lines:

Line 1 passes through the points (0, 5) and (-4, 4).

Line 2 passes through the points (0, -1) and (4, 0).

Let's check if the two lines intersect.

We can do this by substituting the x and y values of one line into the equation of the other line.

For Line 1, substituting (0, 5) into the equation x = 4y + 4:

0 = 4(5) + 4

0 = 20 + 4

0 = 24

The equation is not satisfied, indicating that (0, 5) does not lie on Line 2.

For similar questions on linear equations

https://brainly.com/question/28732353
#SPJ11

Solve please don’t know how to get the answer

Answers

Answer:

5.9 mph

Step-by-step explanation:

The boat's speed is 15 mph

Given the current's speed is x, then

Boat's speed going upstream: 15 - x

=> time going upstream = 130/(15 - x)

Boat's speed going downstream: 15 + x

=> time going downstream = 130/(15 + x)

Total time

130/(15 - x) + 130/(15 + x) = 20.5

130(15 + x) + 130(15 - x ) = 20.5(15 + x)(15 - x)

130(15 + x + 15 - x) = 20.5(225 - x^2)

20.5(225 - x^2) = 130(30)

225 - x^2 = 3900/20.5

x^2 = 225 - 3900/20.5

x = square root of (225 - 3900/20.5)

x = ±5.895 or ±5.9

since speed can't be negative, speed of current is 5.9

Find a unit vector in the direction of AB
, where A(1,2,3) and B(4,5,6) are the given points.

Answers

To find a unit vector in the direction of AB, we need to calculate the vector AB and then normalize it. The vector AB is obtained by subtracting the coordinates of point A from the coordinates of point B: AB = B - A.

AB = (4, 5, 6) - (1, 2, 3) = (3, 3, 3).

To normalize the vector AB, we divide each component of AB by its magnitude. The magnitude of AB can be calculated using the Euclidean norm formula: ||AB|| = √(3^2 + 3^2 + 3^2) = √27 = 3√3.

Now, divide each component of AB by 3√3 to obtain a unit vector in the direction of AB:

(3/3√3, 3/3√3, 3/3√3) = (√3/3, √3/3, √3/3).

Therefore, a unit vector in the direction of AB is (√3/3, √3/3, √3/3).

Learn more about normalize here: brainly.com/question/32234988

#SPJ11

Twenty students in Class A and 20 students in Class B were asked how many hours they took to prepare for an exam. The data sets represent their answers. Class A: {2, 5, 7, 6, 4, 3, 8, 7, 4, 5, 7, 6, 3, 5, 4, 2, 4, 6, 3, 5} Class B: {3, 7, 6, 4, 3, 2, 4, 5, 6, 7, 2, 2, 2, 3, 4, 5, 2, 2, 5, 6} Which statement is true for the data sets?

Answers

Answer:

Step-by-step explanation:

The mean study time of students in Class B is less than students in Class A.

Scarlett left her house at time zero and drove for 2 minutes to the store, at a speed of 8 blocks per minute. Then she stopped and went into the store for 2 minutes. From there, she drove in the same direction at a speed of 3 blocks per minute until she got to the bank, which is 6 blocks away from the store. She stopped at the bank for 6 minutes. Then she drove home at a speed of 2 blocks every minute. Make a graph of showing the number of blocks away from home that Scarlett is � x minutes after she leaves her house, until she gets back home.

Answers

Answer:

The required block diagram that shows how much distance Scarlett is away from the home is shown in the image attached.

Step-by-step explanation:

As given in the question Scarlett left her house at time zero and drove to the store, which is 3 blocks away, at a speed of 1 block per minute.

Then she stopped and went into the store for 4 minutes.

she drove in the identical at a rate of 5 blocks per minute until she got to the bank, which is 15 blocks away from the store.

Here,

1 Approach, Scarlett moves with the speed of a block per minute

Total distance travel = 3 block

Approach 2 Scarlett moves with the speed of 5 blocks per minute for 3 minutes

Total distance travel = 15 block

Approach 3 Scarlett moves with the speed of 3 blocks per minute for 1 minute

Total block traveled = 3 + 15 = 18

Now, Approach 3 is to retrace the path at the rate of 3 blocks per minute,

All these calculations is been shown in the block diagram.

Thus, the required block diagram that shows how much distance Scarlett is away from the home is shown in the image attached.

3/5=
3/3=
Write with the same denominator

Answers

The common denominator for 3/5 and 3/3 is 15.

3/5 = 9/15

3/3 = 15/15

Answer:

Try this

so u look for the common denominator for both which will be 15 the u convert both

3/5=  9/15

3/3=15/15

solve the following system ror y:
2x - 15y = -10
-4x + 5y =-30

a 2
b 10
c 2x-40
d -2

Answers

The solution to the system of equations for y is y = 2. So, the correct answer is (a) 2.

To solve the system of equations for y, we can use the method of substitution or elimination. Let's use the method of elimination:

We have the following system of equations:

2x - 15y = -10

-4x + 5y = -30

To eliminate the x term, we can multiply equation 1 by 2 and equation 2 by 4, so the coefficients of x will cancel out when we add the equations:

4(2x - 15y) = 4(-10) => 8x - 60y = -40

2(-4x + 5y) = 2(-30) => -8x + 10y = -60

Now we can add equations 3 and 4:

(8x - 60y) + (-8x + 10y) = -40 + (-60)

-60y + 10y = -100

-50y = -100

Dividing both sides by -50:

y = (-100)/(-50)

y = 2

Therefore, the solution to the system of equations for y is y = 2.

So, the correct answer is (a) 2.

For more such questions on equations , Visit:

https://brainly.com/question/22688504

#SPJ11

Exam
A
B
C. C
D
F
What else is
needed to prove
these triangles
congruent using
the SAS postulate?
A. Nothing else is needed to use the SAS postulate.
B. ZD = LB.
11
(

Answers

Check the picture below.

If X is uniformly distributed over (0,1) and Y is exponentially distributed with parameter λ = 1, find the distribution of (a) (5 points) Z=X+Y (b) (5 points) Z=X/Y

Answers

a. The distribution of Z=X+Y is fZ(z) = 0

b. The distribution of  Z=X/Y is a constant distribution with fZ(z)

To find the distribution of Z in both cases, we need to use the concept of convolution for the sum of random variables.

(a) Z = X + Y:

Since X is uniformly distributed over (0,1) and Y is exponentially distributed with parameter λ = 1, we can find the distribution of Z by convolving the probability density functions (PDFs) of X and Y.

The PDF of X is a constant function over the interval (0,1) and is given by:

fX(x) = 1, for 0 < x < 1

fX(x) = 0, otherwise

The PDF of Y, being exponentially distributed with parameter λ = 1, is given by:

fY(y) = λ * exp(-λy), for y > 0

fY(y) = 0, otherwise

To find the distribution of Z, we convolve the PDFs of X and Y:

fZ(z) = ∫ fX(z-y) * fY(y) dy

= ∫ 1 * exp(-y) dy, for z-1 < y < z

Integrating the above expression:

fZ(z) = [-exp(-y)] from z-1 to z

= exp(-(z-1)) - exp(-z), for 1 < z < 2

= 0, otherwise

Therefore, the distribution of Z = X + Y is given by:

fZ(z) = exp(-(z-1)) - exp(-z), for 1 < z < 2

fZ(z) = 0, otherwise

(b) Z = X/Y:

To find the distribution of Z, we can use the method of transformation of random variables.

Let's define W = X/Y. We can find the cumulative distribution function (CDF) of W, and then differentiate to obtain the PDF.

The CDF of W can be expressed as:

FZ(z) = P(Z ≤ z) = P(X/Y ≤ z)

To proceed, we'll consider two cases separately:

Case 1: z > 0

In this case, we have:

FZ(z) = P(X/Y ≤ z) = P(X ≤ zY) = ∫[0,1] ∫[0,zy] 1 dy dx

= ∫[0,1] zy dy dx

= z ∫[0,1] y dy dx

= z [y^2/2] from 0 to 1

= z/2

Case 2: z ≤ 0

In this case, we have:

FZ(z) = P(X/Y ≤ z) = P(X ≥ zY) = 1 - P(X < zY) = 1 - ∫[0,1] ∫[0,zy] 1 dy dx

= 1 - ∫[0,1] zy dy dx

= 1 - z ∫[0,1] y dy dx

= 1 - z [y^2/2] from 0 to 1

= 1 - z/2

Therefore, the CDF of Z = X/Y is:

FZ(z) = z/2, for z > 0

FZ(z) = 1 - z/2, for z ≤ 0

Differentiating the CDF, we obtain the PDF:

fZ(z) = 1/2, for z > 0

fZ(z) = 1/2, for z ≤ 0

Therefore, the distribution of Z = X/Y is a constant distribution with fZ(z)

Learn more about distribution at https://brainly.com/question/15707574

#SPJ11

given f (x) = x2 6x 5 and g(x) = x3 x2 − 4x − 4, find the domain of

Answers

The domain of both functions is the set of all real numbers, which can be expressed as (-∞, +∞) or simply as "all real numbers."

To find the domain of the functions f(x) = x^2 - 6x + 5 and g(x) = x^3 + x^2 - 4x - 4, we need to determine the set of all possible values for x for which the functions are defined.

The domain of a function is the set of all real numbers for which the function is defined without any restrictions or division by zero.

For both f(x) and g(x), there are no square roots, fractions, or any other operations that could introduce undefined values. Therefore, the domain of both functions is the set of all real numbers, which can be expressed as (-∞, +∞) or simply as "all real numbers."

For more information on domain visit: brainly.com/question/25199132

#SPJ11

6.2. the joint probability mass function of the random variables x, y, z is p(1, 2, 3) = p(2, 1, 1) = p(2, 2, 1) = p(2, 3, 2) = 1 4 find (a) e[xyz], and (b) e[xy xz yz]

Answers

To calculate the expected values, we need to use the joint probability mass function (PMF) of the random variables.

In this case, we are given the following probabilities:

p(1, 2, 3) = p(2, 1, 1) = p(2, 2, 1) = p(2, 3, 2) = 1/4

(a) To find E[XYZ], we need to calculate the expected value of the product of the three random variables.

E[XYZ] = Σx Σy Σz xyz * p(x, y, z)

Substituting the given probabilities:

E[XYZ] = (123)(1/4) + (211)(1/4) + (221)(1/4) + (232)(1/4)

Simplifying:

E[XYZ] = 6/4 + 2/4 + 4/4 + 12/4

E[XYZ] = 24/4

E[XYZ] = 6

E[XYZ] is equal to 6.

(b) To find E[XY * XZ * YZ], we need to calculate the expected value of the product of the pairwise products of the random variables.

E[XY * XZ * YZ] = Σx Σy Σz xy * xz * yz * p(x, y, z)

Substituting the given probabilities:

E[XY * XZ * YZ] = (12)(13)(23)(1/4) + (21)(23)(13)(1/4) + (22)(21)(21)(1/4) + (23)(22)(32)(1/4)

Simplifying:

E[XY * XZ * YZ] = 666*(1/4) + 1263*(1/4) + 822*(1/4) + 1286*(1/4)

E[XY * XZ * YZ] = 6*(6/4) + 12*(18/4) + 8*(2/4) + 12*(24/4)

E[XY * XZ * YZ] = 36/4 + 216/4 + 16/4 + 288/4

E[XY * XZ * YZ] = 556/4

E[XY * XZ * YZ] = 139

E[XY * XZ * YZ] is equal to 139.

To know more about probability visit:

brainly.com/question/32117953

#SPJ11

1. Evil Simon's billiards. a) Simon gives you a 7-gallon jug and a 5-gallon jug and asks you to make 3 gal- lons of water. Draw the corresponding bil- liards table twice and add to these drawings the paths that the billiards ball takes when launched from the upper left and lower right corners. Spell out the instructions for the shortest solution to Simon's task as in the lecture notes. b) Next, Simon gives you a 12-gallon jug and a 9-gallon jug. Which numbers of gallons (1, 2,..., 12) can you make up with our method? c) Read the part of these lecture notes ded- icated to a graphical method for finding the least common multiple of two integers. Use this method to find the least common mul- tiple of 18 and 10. That is, draw the cor- responding billiards table, draw the path of the billiards ball and then use your drawing to find the least common multiple. d) You have a 4-minute hourglass and a 7- minute hourglass. How can you measure a period of exactly 9 minutes? The hour- glasses must always be running: you cannot lay them on their sides. (Hint: The Die Hard method does not help with this. Just do this one from scratch.)

Answers

a)The two jugs will be known as A (the larger) and B (the smaller). Fill jug A with water and then pour this into jug B until it is full. We know that jug A contains 7 units of water and jug B contains 5 units of water, with 2 units remaining in jug A.

Now pour jug B down the sink and fill it with the 2 units from jug A.

Finally, fill jug A with water and pour it into jug B until it is full.

We now have 3 units of water in jug A and 4 units of water in jug B.

The answer can be expressed in this form as follows:

((A -> B, 7 -> 5), (B -> Sink, 5 -> 0), (A -> B, 2 -> 0), (A -> B, 7 -> 5), (B -> Sink, 5 -> 0), (A -> B, 4 -> 0)). T

he directions are as follows: Start with A full and B empty.

Pour A into B until B is full, pour B away, pour A into B until B is full, pour A into B until B is full, pour B away, pour A into B until B is full.

For this solution, we had to create four states.

b) The following is the least common multiple of 9 and 12: LCM(9, 12) = 36.

The values that can be reached with A = 12 and B = 9 are as follows: 0, 9, 12, 18, 24, 27, and 36.

c) The least common multiple of 10 and 18 can be found using the same process as above, where A is 18 and B is 10.

The following is the least common multiple of 10 and 18: LCM(10, 18) = 90. The values that can be reached with A = 18 and B = 10 are as follows: 0, 10, 18, 20, 30, 36, 40, 45, 50, 54, 60, 70, 72, 80, 81, and 90.

d) This is a bit more complicated.

Flip both hourglasses at the same time and let them run for 4 minutes.

When the 4-minute hourglass is complete, flip it over and let it run again. When it is complete, the 9-minute interval is complete as well.

To know more about Units  visit :

https://brainly.com/question/23843246

#SPJ11

Determine the values of k so that the following linear system of equations (in x, y and z) has:
(i) a unique solution; (ii) no solution; (iii) an infinite number of solutions.
2x + (k + 1)y + 2z = 3
2x + 3y + kz = 3
3x + 3y − 3z = 3

Answers

The values are (i) Unique solution: k ≠ 2

(ii) No solution: k = 2

(iii) Infinite solutions: k = 2

To determine the values of k for the given linear system, we can analyze the coefficient matrix and the augmented matrix.

The coefficient matrix is:

[ 2 (k + 1) 2 ]

[ 2 3 k ]

[ 3 3 -3 ]

We can perform row operations to simplify the matrix:

R2 = R2 - R1

R3 = R3 - R1

The simplified matrix becomes:

[ 2 (k + 1) 2 ]

[ 0 (2 - k) (k - 2) ]

[ 0 (2 - k) (-5) ]

Now, let's analyze the augmented matrix:

[ 2 (k + 1) 2 | 3 ]

[ 0 (2 - k) (k - 2) | 0 ]

[ 0 (2 - k) (-5) | 0 ]

(i) For a unique solution, the coefficient matrix must be non-singular, which means its determinant must be nonzero. Thus, we need to find the values of k for which the determinant of the coefficient matrix is nonzero.

(ii) For no solution, the coefficient matrix and the augmented matrix must have different ranks. So, we need to determine the values of k for which the rank of the coefficient matrix differs from the rank of the augmented matrix.

(iii) For an infinite number of solutions, the coefficient matrix and the augmented matrix must have the same rank, and the rank must be less than the number of variables. Thus, we need to find the values of k for which the rank of both matrices is equal and less than 3.

By analyzing the determinant and ranks, we can determine the values of k for each case.

To know more about linear system of equations refer here:

https://brainly.com/question/20379472

#SPJ11

5 (p - 1) p = 8 whats the answer for it??

Answers

Answer:

p ≈ 0.842 and p ≈ -1.842

Step-by-step explanation:

To solve the equation 5(p - 1)p = 8, we can begin by expanding the expression:

5(p - 1)p = 8

5(p^2 - p) = 8

Distribute the 5:

5p^2 - 5p = 8

Rearrange the equation to bring all terms to one side:

5p^2 - 5p - 8 = 0

Now we have a quadratic equation. To solve it, we can use factoring, completing the square, or the quadratic formula. In this case, factoring is not straightforward, so let's use the quadratic formula:

Given an equation in the form ax^2 + bx + c = 0, the quadratic formula states that the solutions for x are given by:

x = (-b ± √(b^2 - 4ac)) / (2a)

For our equation, a = 5, b = -5, and c = -8. Substituting these values into the quadratic formula, we get:

p = (-(-5) ± √((-5)^2 - 4(5)(-8))) / (2(5))

p = (5 ± √(25 + 160)) / 10

p = (5 ± √185) / 10

The solutions for p are given by p ≈ 0.842 and p ≈ -1.842.

Other Questions
a 44-year-old woman presents to the emergency department complaining of abdominal pain, nausea, and vomiting. the pain began last night and woke her up from sleep approximately six hours ago. the pain is located in the epigastric area and the right upper quadrant of the abdomen and she describes it as a squeezing pain. physical exam reveals a positive murphy sign. what is the most likely diagnosis? were on earth can you find transform boundaries a wire 28.0 cm long lies along the z-axis and carries a current of 8.10 a in the z-direction. the magnetic field is uniform and has components Bx = -0.245 T , By = -0.950 T, and Bz = -0.348 T . What is tribe that moved to england after the huns attacked Consider the vector field F(x, y) = yi + xyj. Then F(2, 1) is equal to: a. 2i +4j O b. O c. 2i +2j O d. 4i +2j O e. 2i + 8j None of these the icd-10-cm contains 71,924 procedure codes. 1. reflection: reflect on at least 2 key concepts you have learned from this simulation. how would you relate it to the physiological functions of the body? Find a polar equation for the curve represented by the given Cartesian equation xy = 2 ,2 = 2 sec(9)sin() | x Write a letter to your friend Inviting to him/her for your inter house sports Competition. Giving him/her three (3) reassos why he/she so should be there. how would a nurse test a patients superficial reflex? boys with authoritarian parents are more likely to show , whereas girls more often show . question 74 options: a. aggressiveness and unruliness; anxiousness and unhappiness b. unhappiness and anxiousness; aggressiveness and unruliness c. personality disorders and low grades; adhd and high grades d. adhd and high grades; personality disorders and low grades Sedimentary cycle having a small gaseous component is found in A) Phosphorus B) Nitrogen C) Carbon D) Sulphur. What is the value of a + c? Explain or show your reasoning.is tangent to the circle and is therefore perpendicular toSo the measure ofis 90 degrees. The angle measures of a triangle add to 180 degrees, so by substitution, wecan determine that a + c =Line find the point of inflection of the graph of the function. (if an answer does not exist, enter dne.) f(x) = x3 6x2 23x 30 two stars of the same spectral class must have the same. true or false G G x + 9x4+x Given, 3 + 2x + 4 using Rouche's thore how to show it has , inside the circle Partners Dennis and Lilly have decided to liquidate their business. The following information is available:Dennis and Lilly share profits and losses in a 3:2 ratio. During the first month of liquidation, half the inventory is sold for $60,000, and $60,000 of the accounts payable is paid. During the second month, the rest of the inventory is sold for $45,000, and the remaining accounts payable are paid. Cash is distributed at the end of each month, and the liquidation is completed at the end of the second month.7. Refer to the information provided above. Using a safe payments schedule, how much cash will be distributed to Dennis at the end of the first month?A. $64,000B. $60,000C. $24,000D. $36,000 A 4 kg steel ball is attached to a vertical spring. It starts a simple harmonic oscillation between a high point A and a low point B that are 20cm apart, with a period of t seconds. a) What is the amplitude of the oscillation? b) Spring constant of the spring? c) Maximum speed? d) Where is the location of the ball when it has the maximum kinetic energy (use A or B as reference points)? when did congress bar slave markets from the nations capital? Frances Willard lobbied for these issues important to woman EXCEPT:a. child-labor laws.b. the right to vote.c. for women to become ministers.d. the eight-hour workday.e. government-funded kindergartens.